All Subjects

2024 AP English Language and Composition Exam Guide

11 min read • june 18, 2024

A Q

Your guide to the 2024 AP English Language and Composition exam

We know that studying for your AP exams can be stressful, but Fiveable has your back! We created a study plan to help you crush your AP English Language and Composition exam. This guide will continue to update with information about the 2024 exams, as well as helpful resources to help you do your best on test day.  Unlock Cram Mode  for access to our cram events—students who have successfully passed their AP exams will answer your questions and guide your last-minute studying LIVE! And don't miss out on unlimited access to our database of thousands of practice questions.

Format of the 2024 AP English Language and Composition exam

This year, all AP exams will cover all units and essay types. The 2024 AP English Language and Composition exam format will be:

  • Section I: Multiple Choice - 45% of your score- - 45 questions in 1 hour
  • Section II: Free Response Section - 55% of your score- - 2 hours and 15 minutes for:- 1 synthesis essay - 1 rhetorical analysis essay - - 1 argument essay

Scoring Rubric for the 2024 AP Lang Essays

  • Synthesis Essay - Thesis - 1 point for a defensible thesis that responds to the prompt- Evidence and Commentary - Max of 4 points for providing evidence from at least 3 sources that support the line of reasoning AND commentary that explains and analyzes the evidence- Sophistication - 1 point any of the following: - Creating a nuanced argument - Showing the limitations of the argument - Making effective rhetorical choices - Employing a style that is vivid and persuasive- Rhetorical Analysis Essay - Thesis - 1 point for a defensible thesis that analyzes rhetorical choices- Evidence and Commentary - Max of 4 points for providing specific evidence AND consistently explaining how the evidence relates to the line of reasoning AND showing how the rhetorical choices contribute to the author's message .- Sophistication - 1 point for any of the following: - Explaining the significance of the rhetorical choices ( rhetorical situation ) - Explaining the complexities of the passage and their purpose - Employing a style that is vivid and persuasive- Argument Essay - Thesis - 1 point for a defensible thesis- Evidence and Commentary - Max of 4 points for providing specific evidence AND consistently explaining the relevance of that evidence.- Sophistication - 1 point for any of the following: - Crafting a nuanced argument by identifying complexities - Explaining the limitations of the argument by placing it in a broader context - Making rhetorical choices to improve the argument - Employing a style that is vivid and persuasive Check out our study plan below to find resources and tools to prepare for your AP English Language and Composition exam.

When is the 2024 AP English Language and Composition Exam and How Do I Take It?

** The exam will be on paper and in-person at your school: Tuesday, May 14, 2024, at 8:00 AM, your local time.  **

How Should I Prepare for the AP Lang Exam?

  • First, take stock of your progress in the course so far. What areas have you excelled and which sections need more focus? Download the  AP English Language Cheatsheet PDF  - a single sheet that covers everything you need to know at a high level. Take note of your strengths and weaknesses!
  • Build your study plan to review every unit and question type, but focus most on the areas that need the most improvement and practice. We’ve put together this plan to help you study between now and May. This will cover all of the units and essay types to prepare you for your exam- - Practice essays are your best friends! The more essays you write, the more automatic the process will come, and the easier the AP exam will be!- - Try some of the past exam questions  [object Object]
  • We've put together the study plan found below to help you study between now and May. This will cover all of the units and essay types to prepare you for your exam. Pay special attention to the units that you need the most improvement in.
  • Study, practice, and review for test day with other students during our live cram sessions via  Cram Mode . Cram live streams will teach, review, and practice important topics from AP courses, college admission tests, and college admission topics. These streams are hosted by experienced students who know what you need to succeed.

Pre-Work: Set Up Your Study Environment

Before you begin studying, take some time to get organized.

🖥 Create a study space.

Make sure you have a designated place at home to study. Somewhere you can keep all of your materials, where you can focus on learning, and where you are comfortable. Spend some time prepping the space with everything you need and you can even let others in the family know that this is your study space. 

📚 Organize your study materials.

Get your notebook, textbook, prep books, or whatever other physical materials you have. Also, create a space for you to keep track of review. Start a new section in your notebook to take notes or start a Google Doc to keep track of your notes. Get yourself set up!

📅 Plan designated times for studying.

The hardest part about studying from home is sticking to a routine. Decide on one hour every day that you can dedicate to studying. This can be any time of the day, whatever works best for you. Set a timer on your phone for that time and really try to stick to it. The routine will help you stay on track.

🏆 Decide on an accountability plan.

How will you hold yourself accountable to this study plan? You may or may not have a teacher or rules set up to help you stay on track, so you need to set some for yourself. First, set your goal. This could be studying for x number of hours or getting through a unit. Then, create a reward for yourself. If you reach your goal, then x. This will help stay focused!

2024 AP Lang Study Guide

🚧 unit 1 foundations of rhetoric: analysis of the rhetorical situation and claims., big takeaways:.

Unit 1 is an introductory unit that lays the foundations for the reading skills associated with how to understand and analyze complex texts. Skills here include identifying the ASPECTS of a text, analyzing the claim given and the evidence used to support that claim, and determining the function of the “chunks” in the argument. Because the content in this unit is very foundational, it is looped throughout the rest of the course instruction.

Definitely do this:

📚 Read these study guides:

  • Unit 1 Overview: Claims, Reasoning, and Evidence
  • 1.1 Identifying the purpose and intended audience of a text
  • 1.2 Examining how evidence supports a claim
  • 1.3 Developing paragraphs as part of an effective argument 🎥 Watch these videos:  

College Board’s Instructional Video:  Overview of The Rhetorical Situation .- Fiveable’s  How to Read Like an AP Student .- Rhetorical Analysis Thesis Statements - Rhetorical Analysis Body Paragraphs ✍️ Practice:

Use the  Fiveable ASPECTS Guidesheet to help you break down a complex text. 🗺 Can you identify these rhetorical devices?

You won’t be asked to name drop on the exam, but it can be helpful to use devices when discussing strategies. Try  this Quizlet to help prepare.

Unit 2 Foundations of Argument: Analysis of an author’s choices in appeals and evidence

Unit 2 is an introductory unit that builds onto the foundations of rhetorical ASPECTS and moves toward planning and writing your own arguments. This unit focuses on the relationships between subject, speaker, and message, including examination of the structure and purpose of the given argument. The unit then moves into the developing thesis statements and building your own arguments with a clear line of reasoning.

  • Unit 2 Overview: Organizing Information for a Specific Audience
  • 2.1 Analyzing audience and its relationship to the purpose of an argument
  • 2.2 Building an argument with relevant and strategic evidence
  • 2.3 Developing thesis statements
  • 2.4 Developing structure and integrating evidence to reflect a line of reasoning 🎥 Watch these videos:  

College Board’s Instructional Video:  Identify Rhetorical Situation in a Pre 20th Century Text . 

Fiveable’s video on  How to Find Rhetorical Devices 📰 Check out these articles:

Here’s a  list of recommended rhetorical devices with definitions and examples! ✍️ Practice:

Use the  Fiveable Rhetorical Precis Guidesheet to help you break down a complex text. 🗺 Can you identify these elements of practical argument?

You won’t be asked to name drop of the exam, but it can be helpful to use devices when discussing strategies. Try  this Quizlet to help prepare. -----------

👥 Unit 3 Confluence: Synthesis of multiple sources in argumentation

Unit 3 approaches multiple perspectives in argument through the lens of synthesis (that’s FRQ 1). In this study, you learn to identify effective and faulty reasoning while integrating a variety of evidence from credible resources that is properly cited in an original text.

  • Unit 3 Overview: Perspectives and How Arguments Relate
  • 3.1 Interpreting character description and perspective
  • 3.2 Identifying and avoiding flawed lines of reasoning
  • 3.3 Introducing and integrating sources and evidence
  • 3.4 Using sufficient evidence for an argument
  • 3.5 Attributing and citing references
  • 3.6 Developing parts of a text with cause-effect and narrative methods 🎥 Watch these videos:

Fiveable’s Introduction into  Synthesis Essays and  How to Begin Your Argument

College Board’s Instructional Video:  Complexity in Argument . 🗺 Can you identify these elements of synthesis?

👀 Unit 4 Reasoning: Analysis of argument from introduction to conclusion

Unit 4 includes a greater depth of focus on the writing of effective arguments -- the line of reasoning created in the introduction, built with modes of discourse, and strengthened in the conclusion. An important note about these skills of argumentation is that they build toward all parts of every FRQ. 

  • Unit 4 Overview: How writers develop arguments, intros, and conclusion
  • 4.1 Developing and connecting thesis statements and lines of reasoning
  • 4.2 Developing introductions and conclusions
  • 4.3 Adjusting an argument to address new evidence 🎥 Watch these videos:  

College Board’s Instructional Video:  Understanding a Line of Reasoning .

Fiveable’s  Effective Annotations . ✍️ Practice:  

Try Fiveable’s  Guide to LOR Body Paragraphs . 🗺 Can you identify the rhetorical modes?

You won’t be asked to name drop them on the exam, but it can be helpful to use devices when discussing strategies. Try  this Quizlet to help prepare. -----------

🧐 Unit 5 Commentary and Analysis: Analysis of complex argument and intentional rhetoric

In Unit 5, the skills look at the minutiae involved in argumentation: development of the line of reasoning that produces strong commentary and maintains the primary claim through all parts of the writing. To achieve these goals, this unit includes a focus on transitions , modifiers , and qualifications for argumentative perspective .  

  • Unit 5 Overview
  • 5.1 Maintaining ideas throughout an argument
  • 5.2 Developing commentary throughout paragraphs
  • 5.3 Using modifiers to qualify an argument and convey perspective
  • 5.4 Using transitions 🎥 Watch these videos:  

Fiveable’s video on  How to Improve Analysis Part 1 and  Part 2 - As well as how to  Embed Quotes into Body Paragraphs - Rhetorical Analysis Body Paragraphs - Synthesis Essay Body Paragraphs - Argument Essay Body Paragraphs 📰 Check out these articles:

Tara Seale’s adaptation for  Creating a Line of Reasoning . ✍️ Practice:

Try Fiveable’s  Guide to LOR Body Paragraphs . ---------

🏃‍♂️ Unit 6 Rhetorical Risks: Analysis of multiple perspectives, bias, and shifts with new evidence

In Unit 6, you will notice a direct link building on the ideas of Unit 3 as this instruction looks at position and perspectives while synthesizing information strategically to support a claim.  For greater depth, this unit moves to modify a current argument to include new evidence.

  • Unit 6 Overview: Position, Perspective, and Bias
  • 6.1 Incorporating multiple perspectives strategically into an argument
  • 6.2 Recognizing and accounting for bias
  • 6.3 Adjusting an argument to new evidence
  • 6.4 Analyzing tone and shifts in tone 🎥 Watch these videos:  

College Board’s Instructional Video:  Creating a Nuanced Argument . 

Fiveable’s video on  Tracking an Author’s Argument ---------

🚀 Unit 7 Complex Argumentation: Analysis of effective arguments, including concession and refutation

The skills of Unit 7 are about putting all units of study together to look at the complexity of a given argument and the effectiveness of the pieces built into that argument.  Though many teachers will have addressed counterarguments, concessions, and refutations before reaching this unit, those skills are highly scrutinized in this segment of learning.

  • Unit 7 Overview: Successful and Unsuccessful Arguments
  • 7.1 Examining complexities in issues
  • 7.2 Considering how words, phrases, and clauses can modify and limit an argument
  • 7.3 Examining how counterargument or alternative perspectives affect an argument
  • 7.4 Exploring how sentence development affects an argument 🎥 Watch these videos:  

Fiveable’s video on  Arguments and Counterarguments

College Board’s Instructional Video:  How Argument Demonstrates Understanding . ✍️ Practice:

Check your progress with  Fiveable’s AP Language Skills Matrix . --------

📝 Unit 8 Style: Analysis of how style influences the audience movement

Unit 8 covers how to understand the influence style has on the audience , and the purpose behind each decision. By analyzing these various tactics, students are able to understand the author’s audience, and how to effectively persuade them. Style is an important part in connecting the rest of the course and understanding how the rhetorical choices and devices are used to accomplish a purpose.

  • Unit 8 Overview: Stylistic Choices
  • 8.1 Choosing comparisons based on an audience
  • 8.2 Considering how sentence development and word choice affect how the writer is perceived by an audience
  • 8.3 Considering how all choices made in an argument affect the audience
  • 8.4 Considering how style affects an argument 🎥 Watch these videos:  

Fiveable’s  Analysis of the Mindset of the Audience - College Board’s Instructional video:  Analyzing and Understanding the Audience 📰 Check out these articles:

College Board’s explanation of  Elements and Context for Style ✍️ Practice:

Review this quizlet on  Elements of Style for more practice. ---------

✏️ Unit 9 Craft: Creation of your own complex argument with synthesis and rhetoric

The final unit of AP Language and Composition covers how to effectively form your own arguments by acknowledging and understanding complexities to create a nuanced and sophisticated argument. It focuses on your ability to comprehend and connect multiple sources to create a well reasoned, and detailed argument as well as how to add in your own rhetorical devices and choices to make your writing more persuasive and effective.   

  • Unit 9 Overview: Developing a Complex Argument
  • 9.1 Strategically conceding, rebutting, or refuting information
  • 9.2 Crafting an argument through stylistic choices like word choice and description 🎥 Watch these videos:  
  • Fiveable’s video on  Creating your own Synthesis Arguments
  • College Board’s video on  Complexities within Arguments and  How to Create a Nuanced Argument

Key Terms to Review ( 38 )

© 2024 fiveable inc. all rights reserved., ap® and sat® are trademarks registered by the college board, which is not affiliated with, and does not endorse this website..

PrepScholar

Choose Your Test

  • Search Blogs By Category
  • College Admissions
  • AP and IB Exams
  • GPA and Coursework

Expert Guide to the AP Language and Composition Exam

author image

Advanced Placement (AP)

article-71342_640.jpg

With the 2023 AP English Language and Composition exam happening on Tuesday, May 9, it's time to make sure that you're familiar with all aspects of the exam. In this article, I'll give a brief overview of the test, do a deeper dive on each of the sections, discuss how the exam is scored, offer some strategies for studying, and finally wrap up with some essential exam day tips.

Exam Overview

The AP Language and Composition exam tests your rhetorical and composition skills. Essentially, how do authors construct effective arguments in their writing? What tools do they use? How can you use those tools to craft effective writing yourself? That is the essence of rhetorical analysis.

The exam has two parts: the first section is an hour-long, 45 question multiple-choice section. It includes five sets of questions, each based on a passage or passages. In this section, there will be 23-25 rhetorical analysis questions which test your rhetorical skills. There will also be 20-22 writing questions which require you to consider revisions to the texts you're shown.

The second section is free response. It starts with a 15-minute reading period, and then you'll have 120 minutes to write three analytical essays:

  • One essay where you synthesize several provided texts to create an argument
  • One essay where you analyze a nonfiction passage for its rhetorical construction
  • One essay where you create an original argument in response to a prompt.

You will have about 40 minutes to write each essay, but no one will prompt you to move from essay to essay—you can structure the 120 minutes as you wish.

In the next sections I'll go over each section of the exam more closely—first multiple choice, and then free response.

The AP English Language and Composition Multiple-Choice

The multiple-choice section tests you on two main areas. The first is how well you can read and understand nonfiction passages for their use of rhetorical devices and tools. The second is how well you can "think like a writer" and make revisions to texts in composition questions.

You will be presented with five passages, about which you will receive a small amount of orienting information, e.g. "This passage is excerpted from a collection of essays on boating" or "This passage is excerpted from an essay written in 19th-century Haiti." Each passage will be followed by a set of questions.

There are, in general, eight question types you can expect to encounter on the multiple-choice section of the exam. I've taken my examples from the sample questions in the " Course and Exam Description ."

eight-1316133_640.jpg

Magic eight-ball says there are eight types of multiple-choice questions!

Type 1: Reading Comprehension

These questions are focused on verifying that you understood what a certain part of the passage was saying on a concrete, literal level. You can identify these questions from phrases like "according to" "refers," etc. The best way to succeed on these questions is to go back and re-read the part of the passage referred to very carefully.

Comprehension.png

Type 2: Implication

These questions take reading comprehension one step further—they are primarily focused on what the author is implying without directly coming out and saying it. These questions will have a correct answer, though, based on evidence from the passage. Which interpretation offered in the answers does the passage most support? You can identify questions like these from words like "best supported," ‘"implies," "suggests," "inferred," and so on.

implies.png

Type 3: Overall Passage and Author Questions

These questions ask about overall elements of the passage or the author, such as the author's attitude on the issue discussed, the purpose of the passage, the passage's overarching style, the audience for the passage, and so on.

You can identify these questions because they won't refer back to a specific moment in the text. For these questions, you'll need to think of the passage from a "bird's-eye view" and consider what all of the small details together are combining to say.

3overall_passage.png

Type 4: Relationships Between Parts of the Text

Some questions will ask you to describe the relationship between two parts of the text, whether they are paragraphs or specific lines. You can identify these because they will usually explicitly ask about the relationship between two identified parts of the text, although sometimes they will instead ask about a relationship implicitly, by saying something like "compared to the rest of the passage."

4relationship.png

Type 5: Interpretation of Imagery/Figurative Language

These questions will ask you about the deeper meaning or implication of figurative language or imagery that is used in the text. Essentially, why did the author choose to use this simile or this metaphor? What is s/he trying to accomplish?

You can generally identify questions like this because the question will specifically reference a moment of figurative language in the text. However, it might not be immediately apparent that the phrase being referenced is figurative, so you may need to go back and look at it in the passage to be sure of what kind of question you are facing.

5imagery.png

Type 6: Purpose of Part of the Text

Still other questions will ask you to identify what purpose a particular part of the text serves in the author's larger argument. What is the author trying to accomplish with the particular moment in the text identified in the question?

You can identify these questions because they will generally explicitly ask what purpose a certain part of the text serves. You may also see words or phrases like "serves to" or "function."

6purpose_of_part.png

Type 7: Rhetorical Strategy

These questions will ask you to identify a rhetorical strategy used by the author. They will often specifically use the phrase "rhetorical strategy," although sometimes you will be able to identify them instead through the answer choices, which offer different rhetorical strategies as possibilities.

7rhetorical_strategy.png

Type 8: Composition

This is the newest question type, first seen in the 2019/2020 school year. For these questions, the student will need to act as though they are the writer and think through different choices writers need to make when writing or revising text.

These questions can involve changing the order of sentences or paragraphs, adding or omitting information to strengthen an argument or improve clarity, making changes to draw reader attention, and other composition-based choices.

body_composition

Some very important stylish effects going on here.

The AP English Language and Composition Free Response

The free response section has a 15-minute reading period. After that time, you will have 120 minutes to write three essays that address three distinct tasks.

Because the first essay involves reading sources, it is suggested that you use the entire 15-minute reading period to read the sources and plan the first essay. However, you may want to glance at the other questions during the reading period so that ideas can percolate in the back of your mind as you work on the first essay.

Essay One: Synthesis

For this essay, you will be briefly oriented on an issue and then given anywhere from six to seven sources that provide various perspectives and information on the issue. You will then need to write an argumentative essay with support from the documents.

If this sounds a lot like a DBQ , as on the history AP exams, that's because it is! However, this essay is much more argumentative in nature—your goal is to persuade, not merely interpret the documents.

Example (documents not included, see 2022 free response questions ):

body-AP-Literature-synthesis

Essay Two: Rhetorical Analysis

In the second essay, you'll be presented with an excerpt from a nonfiction piece that advances an argument and asked to write an essay analyzing the rhetorical strategies used to construct the passage's argument. You will also be given some orienting information—where the passage was excerpted from, who wrote it, its approximate date, where it was published (if at all), and to whom it was directed.

Example (excerpt not included, see 2022 free response questions ):

body-AP-literature-Question-2

Essay Three: Argument

In the third essay, you will be presented with an issue and asked to write a persuasive essay taking a position on the issue. You will need to support your position with evidence from your "reading, experience, and observations."

body-AP-lit-Question-3

This doesn't look like a very well-constructed argument.

How The AP Language and Composition Exam Is Scored

The multiple-choice section of the exam is worth 45% of your score, and the free-response section is worth the other 55%. So each of the three free-response essays is worth about 18% of your score.

As on other APs, your raw score will be converted to a scaled score of 1-5. This exam has a relatively low 5 rate. Only 10% of test takers received a 5 in 2022 , although 56% of students received a score of 3 or higher.

In terms of how the raw score is obtained, the multiple-choice section is similar to other AP multiple-choice sections: you receive a point for every question you answer correctly, and there is no penalty for guessing.

The grading rubrics for the free-response questions were revamped in 2019. They are scored using analytic rubrics instead of holistic rubrics. For each free-response question, you will be given a score from 0-6. The rubrics assess three major areas:

#1: Thesis (0 to 1 points): Is there a thesis, and does it properly respond to the prompt?

#2: Evidence and Commentary (0 to 4 points): Does the essay include supporting evidence and analysis that is relevant, specific, well organized, and supports the thesis?

#3: Sophistication (0 to 1 points): Is the essay well-crafted and does it show a sufficiently nuanced understanding of the prompt?

Each scoring rubric broadly assesses these three factors. However, each task is also different in nature, so the rubrics do have some differences. I'll go over each rubric—and what it really means—for you here.

Synthesis Essay Rubrics

0 For any of the following:
1

EVIDENCE AND COMMENTARY

0
1 AND
2 AND
3 AND
4 AND

SOPHISTICATION

0
1 Responses that earn this point may demonstrate sophistication of thought and/or a complex understanding of the rhetorical situation by doing any of the following:

dough-196235_640.jpg

Time to synthesize this dough into some cookies.

Rhetorical Analysis Essay Rubrics

0
1 AND
2 AND
3 AND AND
4 AND AND

magnifying-glass-914922_640.png

Examine your texts closely!

Argumentative Essay Rubrics

playing-puppies-790638_640.jpg

The best kind of frenzy is a puppy frenzy!

AP English Language Prep Tips

Unlike its cousin, the AP English Literature and Composition exam, the AP Language and Composition exam (and course) have very little to do with fiction or poetry. So some students used to more traditional English classes may be somewhat at a loss as to what to do to prepare.

Luckily for you, I have a whole slate of preparation tips for you!

Read Nonfiction—In a Smart Way

A major thing you can do to prepare for the AP Lang and Comp exam is to read nonfiction— particularly nonfiction that argues a position , whether explicitly (like an op-ed) or implicitly (like many memoirs and personal essays). Read a variety of non-fiction genres and topics, and pay attention to the following:

  • What is the author's argument?
  • What evidence do they use to support their position?
  • What rhetorical techniques and strategies do they use to build their argument?
  • Are they persuasive? What counterarguments can you identify? Do they address them?

Thinking about these questions with all the reading you do will help you hone your rhetorical analysis skills.

Learn Rhetorical Terms and Strategies

Of course, if you're going to be analyzing the nonfiction works you read for their rhetorical techniques and strategies, you need to know what those are! You should learn a robust stable of rhetorical terms from your teacher, but here's my guide to the most important AP Language and Composition terms .

  • We've compiled a list of 20 rhetorical devices you should know.
  • A heroic individual from Riverside schools in Ohio uploaded this aggressively comprehensive list of rhetorical terms with examples. It's 27 pages long, and you definitely shouldn't expect to know all of these for the exam, but it's a useful resource for learning some new terms.
  • Another great resource for learning about rhetorical analysis and how rhetorical devices are actually used is the YouTube Channel Teach Argument , which has videos rhetorically analyzing everything from Taylor Swift music videos to Super Bowl commercials. It's a fun way to think about rhetorical devices and get familiar with argumentative structures.
  • Finally, a great book—which you might already use in your class—is " They Say, I Say. " This book provides an overview of rhetoric specifically for academic purposes, which will serve you well for AP preparation and beyond.

You also need to practice argumentative and persuasive writing. In particular, you should practice the writing styles that will be tested on the exam: synthesizing your own argument based on multiple outside sources, rhetorically analyzing another piece of writing in-depth, and creating a completely original argument based on your own evidence and experience.

You should be doing lots of writing assignments in your AP class to prepare, but thoughtful, additional writing will help. You don't necessarily need to turn all of the practice writing you do into polished pieces, either—just writing for yourself, while trying to address some of these tasks, will give you a low-pressure way to try out different rhetorical structures and argumentative moves, as well as practicing things like organization and developing your own writing style.

once-upon-a-time-719174_640.jpg

Not the most auspicious start to an argumentative essay.

Practice for the Exam

Finally, you'll need to practice specifically for the exam format. There are sample multiple-choice questions in the " AP Course and Exam Description ," and old free-response questions on the College Board website.

Unfortunately, the College Board hasn't officially released any complete exams from previous years for the AP English Language and Composition exam, but you might be able to find some that teachers have uploaded to school websites and so on by Googling "AP Language complete released exams." I also have a guide to AP Language and Composition practice tests .

Once you're prepped and ready to go, how can you do your best on the test?

Looking for help studying for your AP exam? Our one-on-one online AP tutoring services can help you prepare for your AP exams. Get matched with a top tutor who got a high score on the exam you're studying for!

AP Language and Composition Test Day Tips

Here are four key tips for test-day success.

board-1193334_640.jpg

You are one hundred percent success!

Interact With the Text

When you are reading passages, both on the multiple-choice section and for the first two free-response questions, interact with the text! Mark it up for things that seem important, devices you notice, the author's argument, and anything else that seems important to the rhetorical construction of the text. This will help you engage with the text and make it easier to answer questions or write an essay about the passage.

Think About Every Text's Overarching Purpose and Argument

Similarly, with every passage you read, consider the author's overarching purpose and argument. If you can confidently figure out what the author's primary assertion is, it will be easier to trace how all of the other aspects of the text play into the author's main point.

Plan Your Essays

The single most important thing you can do for yourself on the free-response section of the AP English Language exam is to spend a few minutes planning and outlining your essays before you start to write them.

Unlike on some other exams, where the content is the most important aspect of the essay, on the AP Language Exam, organization, a well-developed argument, and strong evidence are all critical to strong essay scores. An outline will help you with all of these things. You'll be able to make sure each part of your argument is logical, has sufficient evidence, and that your paragraphs are arranged in a way that is clear and flows well.

Anticipate and Address Counterarguments

Another thing you can do to give your free responses an extra boost is to identify counterarguments to your position and address them within your essay. This not only helps shore up your own position, but it's also a fairly sophisticated move in a timed essay that will win you kudos with AP graders.

envelope-392962_640.jpg

Address counterarguments properly or they might get returned to sender!

Key Takeaways

The AP Language and Composition exam tests your rhetorical skills. The exam has two sections.

The first section is an hour-long, 45 question multiple-choice test based on the rhetorical techniques and composition choices.

The second section is a two-hour free-response section (with a 15-minute initial reading period) with three essay questions: one where you must synthesize given sources to make an original argument, one where you must rhetorically analyze a given passage, and one where you must create a wholly original argument about an issue with no outside sources given.

You'll receive one point for every correct answer on the multiple-choice section of the exam, which is worth 45% of your score. The free-response section is worth 55% of your score. For each free-response question, you'll get a score based on a rubric from 0-6. Your total raw score will be converted to a scaled score from 1-5.

Here are some test prep strategies for AP Lang:

#1 : Read nonfiction with an eye for rhetoric #2 : Learn rhetorical strategies and techniques #3 : Practice writing to deploy rhetorical skills #4 : Practice for the exam!

Here are some test-day success tips:

#1 : Interact with each passage you encounter! #2 : Consider every text's overarching purpose and argument. #3 : Keep track of time #4 : Plan your essays #5 : Identify and address counterarguments in your essays.

With all of this knowledge, you're ready to slay the AP English Language and Composition beast!

animal-1299749_640.png

Noble knight, prepare to slay the AP dragon!

What's Next?

Want more AP Lang review? We have a complete collection of released AP Language practice tests , as well as a list of the AP Lang terms you need to know and a guide to the multiple choice section .

Taking the AP Literature exam? Check out our ultimate guide to the AP English Literature test and our list of AP Literature practice tests .

Taking other AP exams? See our Ultimate Guides to AP World History , AP US History , AP Chemistry , AP Biology , AP World History , and AP Human Geography .

Need more AP prep guidance? Check out how to study for AP exams and how to find AP practice tests .

Want to build the best possible college application?   We can help.   PrepScholar Admissions combines world-class admissions counselors with our data-driven, proprietary admissions strategies. We've guided thousands of students to get into their top choice schools, from state colleges to the Ivy League. We know what kinds of students colleges want to admit and are driven to get you admitted to your dream schools. Learn more about PrepScholar Admissions to maximize your chance of getting in:

These recommendations are based solely on our knowledge and experience. If you purchase an item through one of our links, PrepScholar may receive a commission.

Trending Now

How to Get Into Harvard and the Ivy League

How to Get a Perfect 4.0 GPA

How to Write an Amazing College Essay

What Exactly Are Colleges Looking For?

ACT vs. SAT: Which Test Should You Take?

When should you take the SAT or ACT?

Get Your Free

PrepScholar

Find Your Target SAT Score

Free Complete Official SAT Practice Tests

How to Get a Perfect SAT Score, by an Expert Full Scorer

Score 800 on SAT Math

Score 800 on SAT Reading and Writing

How to Improve Your Low SAT Score

Score 600 on SAT Math

Score 600 on SAT Reading and Writing

Find Your Target ACT Score

Complete Official Free ACT Practice Tests

How to Get a Perfect ACT Score, by a 36 Full Scorer

Get a 36 on ACT English

Get a 36 on ACT Math

Get a 36 on ACT Reading

Get a 36 on ACT Science

How to Improve Your Low ACT Score

Get a 24 on ACT English

Get a 24 on ACT Math

Get a 24 on ACT Reading

Get a 24 on ACT Science

Stay Informed

Get the latest articles and test prep tips!

Follow us on Facebook (icon)

Ellen has extensive education mentorship experience and is deeply committed to helping students succeed in all areas of life. She received a BA from Harvard in Folklore and Mythology and is currently pursuing graduate studies at Columbia University.

Ask a Question Below

Have any questions about this article or other topics? Ask below and we'll reply!

This site uses various technologies, as described in our Privacy Policy, for personalization, measuring website use/performance, and targeted advertising, which may include storing and sharing information about your site visit with third parties. By continuing to use this website you consent to our Privacy Policy and Terms of Use .

We are experiencing sporadically slow performance in our online tools, which you may notice when working in your dashboard. Our team is fully engaged and actively working to improve your online experience. If you are experiencing a connectivity issue, we recommend you try again in 10-15 minutes. We will update this space when the issue is resolved.

Enter your email to unlock an extra $25 off an SAT or ACT program!

By submitting my email address. i certify that i am 13 years of age or older, agree to recieve marketing email messages from the princeton review, and agree to terms of use., guide to the ap english language and composition exam.

AP English Language Exam

Can you apply the rhetorical triangle to a piece of writing? Are you able to argue a position? The AP ® English Language and Composition exam tests topics and skills discussed in your Advanced Placement English Language course. If you score high enough, your AP English Language score could earn you college credit!

Check out our AP English Language Guide for what you need to know about the exam:

  • Exam Overview
  • Sections and Question Types
  • How to Prepare

What’s on the AP English Language & Composition Exam?

The College Board is very detailed in what they require your AP teacher to cover in his or her AP English Language & Composition course. The exam tests your abilities to understand how authors use rhetoric and language to convey their purpose. Students are also expected to apply these techniques to their own writing and research projects. Some of the major skills tested include the ability to:

  • Identify an author’s purpose and intended audience
  • Recognize rhetorical devices and strategies in an author’s work
  • Demonstrate understanding of citations in research papers
  • Apply these skills and techniques to their own writing
  • Create and organize an argument defended with evidence and reasoning
  • Plan, write, and revise cogent, well-written essays

Check out our line of AP guides  for a comprehensive content review.

AP English Language Sections & Question Types

The AP English Language & Composition exam is 3 hours and 15 minutes long and consists of two sections: a multiple-choice section and a free response section.

Section 1

60 minutes

45 multiple choice questions

45%

Section 2

2 hours and 15 minutes

3 free response questions

55%

Read More: Review for the exam with our  AP English Language Crash Course 

Multiple-Choice

For AP English Language multiple-choice questions, you are presented with two Reading Passages and three Writing passages. The two Reading passages are nonfiction passages taken from all sorts of works. The idea is to get you to focus on rhetorical devices, figures of speech and intended purposes, under rigid time constraints and with material you haven’t seen before. The three Writing passages are student-produced essays. The idea is to get you to revise the essay that help the writer accomplish his or her goal.

Free Response

The AP English Language section contains three essay prompts: a synthesis essay, a rhetorical analysis essay, and an argument essay.

  • Synthesis essay: You’ll be given a scenario and tasked with writing a response using at least three of six or seven short accompanying sources for support.
  • Rhetorical analysis essay: Asks you to analyze the techniques an author uses, and discuss how they contribute to the author’s purpose.
  • Argument essay: Presents a claim or assertion in the prompt and then asks you to argue a position based on your own knowledge, experience, or reading.

How to Interpret AP English Language Scores

AP scores are reported from 1 to 5. Colleges are generally looking for a 4 or 5 on the AP English Language exam, but some may grant AP credit for a 3. Each test is curved so scores vary from year to year. Here’s how AP English Lang students scored on the May 2022 test:

5

Extremely qualified

10.4%

4

Well qualified

21.1%

3

Qualified

24.2%

2

Possibly qualified

29.8%

1

No recommendation

14.5%

Source: College Board

How can I prepare?

AP classes are great, but for many students they’re not enough! For a thorough review of AP English Language content and strategy, pick the AP prep option that works best for your goals and learning style.

  • AP Exams  

Explore Colleges For You

Explore Colleges For You

Connect with our featured colleges to find schools that both match your interests and are looking for students like you.

Career Quiz

Career Quiz

Take our short quiz to learn which is the right career for you.

Connect With College Coaches

Get Started on Athletic Scholarships & Recruiting!

Join athletes who were discovered, recruited & often received scholarships after connecting with NCSA's 42,000 strong network of coaches.

Best 389 Colleges

Best 389 Colleges

165,000 students rate everything from their professors to their campus social scene.

SAT Prep Courses

1400+ course, act prep courses, free sat practice test & events,  1-800-2review, free digital sat prep try our self-paced plus program - for free, get a 14 day trial.

ap lang tips for essays

Free MCAT Practice Test

I already know my score.

ap lang tips for essays

MCAT Self-Paced 14-Day Free Trial

ap lang tips for essays

Enrollment Advisor

1-800-2REVIEW (800-273-8439) ext. 1

1-877-LEARN-30

Mon-Fri 9AM-10PM ET

Sat-Sun 9AM-8PM ET

Student Support

1-800-2REVIEW (800-273-8439) ext. 2

Mon-Fri 9AM-9PM ET

Sat-Sun 8:30AM-5PM ET

Partnerships

  • Teach or Tutor for Us

College Readiness

International

Advertising

Affiliate/Other

  • Enrollment Terms & Conditions
  • Accessibility
  • Cigna Medical Transparency in Coverage

Register Book

Local Offices: Mon-Fri 9AM-6PM

  • SAT Subject Tests

Academic Subjects

  • Social Studies

Find the Right College

  • College Rankings
  • College Advice
  • Applying to College
  • Financial Aid

School & District Partnerships

  • Professional Development
  • Advice Articles
  • Private Tutoring
  • Mobile Apps
  • International Offices
  • Work for Us
  • Affiliate Program
  • Partner with Us
  • Advertise with Us
  • International Partnerships
  • Our Guarantees
  • Accessibility – Canada

Privacy Policy | CA Privacy Notice | Do Not Sell or Share My Personal Information | Your Opt-Out Rights | Terms of Use | Site Map

©2024 TPR Education IP Holdings, LLC. All Rights Reserved. The Princeton Review is not affiliated with Princeton University

TPR Education, LLC (doing business as “The Princeton Review”) is controlled by Primavera Holdings Limited, a firm owned by Chinese nationals with a principal place of business in Hong Kong, China.

AP English Language and Composition

Review the free-response questions from the 2024 ap exam, new for 2024-25: mcqs will have four answer choices.

Starting in the 2024-25 school year, AP English Language and Composition multiple-choice questions (MCQs) will have four answer choices instead of five. This change will take effect with the 2025 exam. All resources have been updated to reflect this change.

Exam Overview

Exam questions assess the course concepts and skills outlined in the course framework. For more information, download the  AP English Language and Composition Course and Exam Description (.pdf)  (CED).

Encourage your students to visit the  AP English Language and Composition student page  for exam information.

Wed, May 14, 2025

AP English Language and Composition Exam

Exam format.

The AP English Language and Composition Exam has question types and point values that stay consistent from year to year, so you and your students know what to expect on exam day.

Section I: Multiple Choice

45 Questions | 1 hour | 45% of Exam Score

  • 23–25 Reading questions that ask students to read and analyze nonfiction texts.
  • 20–22 Writing questions that ask students to “read like a writer” and consider revisions to stimulus texts.

Section II: Free Response

3 Questions | 2 hours 15 minutes (includes a 15-minute reading period | 55% of Exam Score

  • Synthesis Question:  After reading 6 texts about a topic (including visual and quantitative sources), students will compose an argument that combines and cites at least 3 of the sources to support their thesis.
  • Rhetorical Analysis:  Students will read a nonfiction text and analyze how the writer’s language choices contribute to the intended meaning and purpose of the text.
  • Argument:  Students will create an evidence-based argument that responds to a given topic.

Exam Questions and Scoring Information

Ap english language and composition exam questions and scoring information.

View free-response questions and scoring information from past exams.

Score Reporting

Ap score reports for educators.

Access your score reports.

ap lang tips for essays

Expert Guide to the AP Language and Composition Exam

ap lang tips for essays

The AP Language and Composition Exam is a comprehensive assessment of students' reading, writing, and critical thinking skills. Here is an expert guide to help you navigate and excel in this exam:

1. Exam Format: The AP Language and Composition Exam consists of multiple-choice questions and free-response tasks. The multiple-choice section tests your reading comprehension and analysis skills, while the free-response section assesses your ability to write coherent and persuasive essays.

2. Analyzing Rhetorical Strategies: A key focus of the exam is analyzing and understanding rhetorical strategies used in various texts. This includes identifying and evaluating techniques such as ethos, pathos, logos, and rhetorical devices like imagery, figurative language, and tone. Practice analyzing different types of texts, including speeches, articles, essays, and advertisements.

3. Essay Writing Skills: The free-response section requires you to write three essays: a synthesis essay, a rhetorical analysis essay, and an argument essay. Develop strong essay writing skills, including thesis development, evidence selection, and paragraph organization. Practice constructing well-structured, coherent, and persuasive arguments within the given time constraints.

4. Close Reading and Annotation: Effective close reading and annotation skills are crucial for success in the exam. Learn to identify the main ideas, key details, and rhetorical elements in the provided passages. Annotate the text to mark important points, make connections, and track your understanding of the author's purpose and argument.

5. Vocabulary and Grammar: Enhance your vocabulary and grammar skills to express your ideas clearly and precisely. Use varied and appropriate language to convey your analysis and arguments effectively. Pay attention to sentence structure, punctuation, and word choice to ensure coherence and precision in your writing.

6. Practice and Timed Mock Exams: Regular practice is essential to build your skills and confidence. Take timed mock exams to simulate the exam conditions and develop your time management skills. Review your performance, identify areas for improvement, and seek feedback from teachers or peers.

7. Read Widely: Expand your reading repertoire by engaging with diverse texts from different genres and time periods. Reading extensively will improve your comprehension, vocabulary, and ability to recognize different writing styles and rhetorical strategies.

8. Critical Thinking and Analysis: Develop your critical thinking skills by analyzing the effectiveness of arguments, evaluating evidence, and recognizing biases and logical fallacies. Practice constructing well-reasoned arguments and counterarguments to strengthen your analysis.

9. Stay Updated with Current Events: Stay informed about current events and societal issues as they often form the basis of essay prompts and analysis passages. Familiarize yourself with contemporary debates, social, and political issues, and be prepared to apply your knowledge to the exam questions.

10. Seek Resources and Guidance: Utilize available resources, such as study guides, practice exams, and online resources, to enhance your preparation. Seek guidance from teachers, tutors, or peers to clarify any doubts and improve your understanding of the exam requirements.

The AP English Language and Composition Multiple-Choice

The multiple-choice section of the AP English Language and Composition exam assesses your reading comprehension and analysis skills. Here are some key points to understand and excel in this section:

1. Format and Structure: The multiple-choice section consists of a series of passages followed by a set of questions. The passages can include a variety of genres such as essays, speeches, articles, and excerpts from books or plays. Each passage is accompanied by multiple-choice questions that require you to analyze the author's purpose, rhetoric, and style.

2. Close Reading: Effective close reading is crucial for success in the multiple-choice section. Read the passages carefully, paying attention to details, tone, and the author's use of rhetorical devices. Underline or annotate important sections to help you remember key points and refer back to them when answering the questions.

3. Understanding Rhetorical Devices: Familiarize yourself with common rhetorical devices such as ethos, pathos, logos, irony, figurative language, and tone. These devices are frequently used by authors to convey their message and persuade the reader. Be prepared to identify and analyze how these devices contribute to the author's overall argument or purpose.

4. Analyzing Text Structure: Pay attention to the structure of the passages, including the organization of ideas, transitions, and the use of evidence. Identify the main idea, supporting details, and the logical flow of the author's argument. Understanding the structure of the passage will help you answer questions related to the author's intent and the development of their ideas.

5. Answering Strategies: Develop effective strategies for approaching multiple-choice questions. Read each question carefully, making sure to consider all the answer choices before selecting the best option. Pay attention to qualifiers such as "most likely," "least likely," "best supports," etc. Eliminate clearly incorrect choices and make an educated guess if you are unsure.

6. Time Management: The multiple-choice section is timed, so it is important to manage your time effectively. Pace yourself and allocate a specific amount of time for each passage and its corresponding questions. If you encounter a challenging question, mark it and move on, returning to it later if time permits.

7. Practice with Sample Questions: Familiarize yourself with the types of questions commonly asked in the AP English Language and Composition exam by practicing with sample questions. This will help you become more comfortable with the format and style of the questions and improve your ability to identify key elements in the passages.

8. Review Test-Taking Strategies: In addition to content knowledge, review general test-taking strategies that can improve your performance. This includes strategies for eliminating answer choices, using process of elimination, and managing your time effectively.

The AP English Language and Composition Free Response

The free response section of the AP English Language and Composition exam is designed to assess your ability to analyze and respond to rhetorical prompts effectively. Here are some key points to understand and excel in this section:

1. Format and Structure:

The free response section consists of three essay prompts: a synthesis essay, a rhetorical analysis essay, and an argument essay. Each prompt presents you with a specific task and requires you to analyze and respond to a given passage or passages.

2. Synthesis Essay:

In this essay, you are asked to combine information from multiple sources to create a coherent and well-supported argument. You must demonstrate your ability to understand and synthesize different perspectives on a given topic. It is important to analyze the sources critically, identify their main arguments, and use evidence from the sources to support your own argument.

3. Rhetorical Analysis Essay:

In this essay, you are required to analyze the rhetorical strategies employed by the author of a given passage. You need to identify and explain the author's use of rhetorical devices, such as ethos, pathos, logos, figurative language, and tone. Your analysis should focus on how these devices contribute to the author's overall argument and purpose.

4. Argument Essay:

In this essay, you are expected to construct and support your own argument on a given topic. You must develop a clear and coherent thesis statement, provide relevant evidence, and effectively address counterarguments. It is important to use persuasive techniques and rhetorical devices to strengthen your argument.

5. Organization and Structure:

Structure your essays in a clear and logical manner. Each essay should have an introduction that presents your thesis statement, body paragraphs that support your thesis with evidence and analysis, and a conclusion that summarizes your main points and reinforces your argument. Use topic sentences and transitions to ensure a smooth flow of ideas.

6. Evidence and Analysis:

Support your claims and arguments with evidence from the given passages or external sources. Use specific examples, quotes, and references to demonstrate your understanding and provide strong evidence for your analysis. Avoid making unsupported generalizations or relying solely on personal opinions.

7. Time Management:

The free response section is time-limited, so it is crucial to manage your time effectively. Allocate a specific amount of time for each essay and stick to it. Leave some time at the end to review and revise your essays for clarity, coherence, and grammatical correctness.

8. Practice and Preparation:

Familiarize yourself with the expectations and requirements of each essay type by practicing with past exam prompts and sample essays. Pay attention to the scoring guidelines provided by the College Board to understand how your essays will be evaluated. Seek feedback from teachers or peers to improve your writing skills and address any weaknesses.

AP English Language Prep Tips

Preparing for the AP English Language exam requires a strategic approach to enhance your reading, writing, and analytical skills. Here are some detailed tips to help you excel in your preparation:

1. Read Widely:

Develop a habit of reading a variety of texts, including fiction, non-fiction, essays, newspaper articles, and editorials. This will expose you to different writing styles, perspectives, and rhetorical devices. Pay attention to the author's tone, purpose, and argumentative strategies.

2. Analyze Rhetorical Devices:

Familiarize yourself with common rhetorical devices such as ethos, pathos, logos, figurative language, and rhetorical appeals. Practice identifying these devices in various texts and analyze how they contribute to the author's message and overall effectiveness.

3. Expand Vocabulary:

Enhance your vocabulary by reading challenging texts and keeping a vocabulary notebook. Learn new words, their definitions, and how they are used in context. Utilize these words in your writing to demonstrate a strong command of language.

4. Practice Timed Writing:

Time yourself while writing essays to simulate the exam conditions. Aim to complete essays within the time limit while maintaining clarity and coherence. Practice different essay types, such as synthesis, rhetorical analysis, and argument essays, to strengthen your skills in each area.

5. Read Sample Essays:

Study well-written sample essays from previous AP exams to understand the expectations and scoring criteria. Analyze their structure, use of evidence, and clarity of argument. Take note of effective introductions, strong thesis statements, and well-supported analysis.

6. Develop Writing Strategies:

Learn to effectively structure your essays with clear introductions, body paragraphs, and conclusions. Use topic sentences, transitions, and evidence to support your claims. Craft strong thesis statements that clearly state your position and guide your essay.

7. Analyze Visual Texts:

Practice analyzing visual texts such as graphs, charts, and images. Understand how visual elements convey information, make arguments, and support claims. Pay attention to the intended audience and the overall impact of visual texts.

8. Practice Multiple-Choice Questions:

Regularly practice multiple-choice questions to improve your reading comprehension and analysis skills. Read passages carefully, annotate as you go, and answer questions based on the given information. Pay attention to details, context, and authorial intent.

9. Seek Feedback:

Share your essays with teachers or peers and seek constructive feedback. Learn from their suggestions to improve your writing skills and address any weaknesses. Consider joining or forming study groups to discuss and analyze different texts and essay prompts.

10. Review Grammar and Mechanics:

Brush up on grammar rules and punctuation to ensure your writing is clear and error-free. Pay attention to sentence structure, verb tense, subject-verb agreement, and pronoun usage. A strong command of grammar enhances the clarity and effectiveness of your writing.

Remember that consistent practice, focused study, and critical reading are key to success in the AP English Language exam. Develop a study schedule, allocate time for reading and writing practice, and stay disciplined in your preparation. With dedication and effort, you can improve your skills and perform well on the exam.

AP Language and Composition Test Day Tips

On the day of the AP Language and Composition exam, it's important to be well-prepared and approach the test with confidence. Here are some detailed tips to help you make the most of your test day:

1. Get a Good Night's Sleep:

Ensure you have a restful night's sleep before the exam day. Being well-rested will help you stay focused and maintain mental clarity throughout the test.

2. Eat a Nutritious Breakfast:

Start your day with a healthy and balanced breakfast. Fueling your body with nutritious food will provide you with the energy you need for the duration of the exam.

3. Arrive Early:

Plan to arrive at the exam location early to avoid any unnecessary stress. Familiarize yourself with the exam venue and locate your assigned room beforehand.

4. Bring Necessary Materials:

Double-check that you have all the required materials for the exam, such as your admission ticket, identification, pens, pencils, erasers, and a watch to keep track of time. Be aware of any specific items allowed or prohibited by the testing guidelines.

5. Read Instructions Carefully:

Take the time to carefully read the instructions provided on the exam booklet and answer sheet. Understand the format, timing, and specific requirements for each section of the test.

6. Pace Yourself:

Time management is crucial in the AP Language and Composition exam. Allocate your time wisely, making sure to complete each section within the specified time limits. Pace yourself and avoid spending too much time on any single question or passage

7. Skim the Questions First:

Before diving into the reading passages, quickly skim the multiple-choice questions to get a sense of what to look for as you read. This can help you focus your attention and save time while reading and analyzing the passages.

8. Read Actively and Annotate:

As you read the passages, actively engage with the text. Underline key points, annotate important details, and mark passages that you may want to refer back to later. This will help you remember crucial information and facilitate your analysis.

9. Plan Your Essays:

For the essay sections, take a few minutes to plan your response before writing. Outline your main points, supporting evidence, and a clear thesis statement. This will provide structure to your essay and ensure a more coherent and organized response.

10. Review Your Work:

If time permits, take a moment to review your answers before submitting your exam. Check for any errors or incomplete responses, and make any necessary corrections or additions. Ensure that you have followed the instructions and provided clear and concise answers.

11. Stay Calm and Focused:

Throughout the exam, maintain a calm and focused mindset. Manage test anxiety by taking deep breaths, maintaining a positive attitude, and focusing on the task at hand. Remember that you have prepared for this exam and trust in your abilities.

12. Follow Exam Regulations:

Adhere to the exam regulations and guidelines provided by the College Board. Maintain academic integrity by refraining from any prohibited behavior, such as cheating or using unauthorized materials.

By following these tips, you can approach the AP Language and Composition exam with confidence and maximize your chances of success. Remember to stay calm, trust your preparation, and showcase your skills in analyzing and responding to complex texts. Good luck!

In conclusion, the AP Language and Composition exam can seem challenging, but with the right preparation and approach, you can excel. Understanding the exam format, practicing multiple-choice questions, mastering the free response section, and developing strong analytical and writing skills are essential for success. Additionally, following test day tips and maintaining a calm and focused mindset will help you perform at your best. By leveraging these insights and strategies, you can navigate the AP Language and Composition exam with confidence and achieve a high score. Good luck on your exam!

You Might Also Like

ap lang tips for essays

Brainstorming for College Essays

This Article is intended to help you brainstorm and begin writing your personal statement essay and all the other college essays. This is a key step to write persuasive college essays

ap lang tips for essays

How to Stand Out through Extracurricular Activities

Do you know the importance of extracurricular activities? Why should you participate in extracurricular activities & how to stand out through it

ap lang tips for essays

A Brief Introduction to College Scholarships

Do you know how to win college scholarship? Know detail information about college scholarship & how to get scholarship successfully? - Read our blog carefully

AP Guru has been helping students since 2010 gain admissions to their dream universities by helping them in their college admissions and SAT and ACT Prep

Free Resources

What are your chances of acceptance?

Calculate for all schools, your chance of acceptance.

Duke University

Your chancing factors

Extracurriculars.

ap lang tips for essays

How to Write the AP Lang Rhetorical Essay

Do you know how to improve your profile for college applications.

See how your profile ranks among thousands of other students using CollegeVine. Calculate your chances at your dream schools and learn what areas you need to improve right now — it only takes 3 minutes and it's 100% free.

Show me what areas I need to improve

What’s Covered:

What is the ap lang rhetorical essay, tips for writing the ap lang rhetorical essay.

  • AP Lang Rhetorical Essay Example

How Will AP Scores Affect College Chances?

The AP English Language Exam is one of the most common AP exams you can take. However, the average score on the exam in 2020 was a 2.96 out of 5. While this may seem a bit low, it is important to note that over 550,000 students take the exam annually. With some preparation and knowing how to study, it is totally possible to do well on this AP exam.

The AP Lang Rhetorical Essay is one section of the AP English Language Exam. The exam itself is 3 hours and 15 minutes long, and is broken into two sections. The first part of the exam is a 60 minute, 45-question multiple-choice section. The questions on this part of the exam will test your ability to read a passage and then interpret its meaning, style, and overall themes. After the multiple-choice section, there is a section lasting 2 hours and 15 minutes with three “free response” essays. This includes the synthesis essay, the rhetorical analysis essay, and the argument essay. 

  • In the synthesis essay , you will have to develop an argument using pieces of evidence provided to you. 
  • The argumentative essay will have you pick a side in a debate and argue for or against it.
  • The rhetorical essay requires that you discuss how an author’s written passage contributes to a greater meaning or theme. 

The rhetorical essay is perhaps the most unique of all AP Lang exam essays because it requires the test taker to analyze and interpret the deeper meanings of the passage and connect them to the author’s writing style and writing syntax in only 40 minutes. This essay can be the trickiest because it requires you to have knowledge of rhetorical strategies and then apply them to a passage you’ve never seen before.

1. Outline Your Essay Before Writing

One of the most important parts of the AP Lang essays is structuring your essay so that it makes sense to the reader. This is just as important as having good content. For this essay in particular, you’ll want to read the passage first and write a brief outline of your points before you begin the essay. This is because you will want to write the essay using the passage chronologically, which will be discussed in detail below.

2. Understand Rhetorical Strategies 

If you feel like you don’t know where to start as you prepare to study for the rhetorical essay portion of the exam, you aren’t alone. It is imperative that you have a grasp on what rhetorical strategies are and how you can use them in your essay. One definition of rhetoric is “language carefully chosen and arranged for maximum effect.” This can include types of figurative language (metaphor, simile, personification, pun, irony, etc.) elements of syntax (parallelism, juxtaposition, anthesis, anaphora, etc), logical fallacies, or persuasive appeals. Overall, there are many elements that you can analyze in an essay and having a good grasp on them through practice and memorization is important.

3. Keep the Essay Well Structured 

Even if you understand the various rhetorical strategies you can use, where do you begin? First of all, you’ll want to write a strong introduction that outlines the purpose of the piece. At the end of this introduction, you will write a thesis statement that encapsulates all the rhetorical strategies you discuss. Perhaps these are style elements, tone, or syntax. Be sure to be specific as you list these.

Next, you will create your body paragraphs. As you discuss the rhetorical elements in the piece and tie them back to the work’s meanings, be sure to discuss the points in chronological order. You don’t have to discuss every single strategy, but just pick the ones that are most important. Be sure to cite the line where you found the example. At the end of the essay, write a short conclusion that summarizes the major points above.

4. Be Sure to Explain Your Examples

As you write the essay, don’t just list out your examples and say something like “this is an example of ethos, logos, pathos.” Instead, analyze how the example shows that rhetoric device and how it helps the author further their argument. As you write the rhetorical essay, you’ll want to be as specific and detail-focused as possible. 

ap lang tips for essays

Discover your chances at hundreds of schools

Our free chancing engine takes into account your history, background, test scores, and extracurricular activities to show you your real chances of admission—and how to improve them.

AP Lang Rhetorical Analysis Essay Example

Below is a prompt and example for a rhetorical essay, along with its score and what the writer did well and could have improved:

The passage below is an excerpt from “On the Want of Money,” an essay written by nineteenth-century author William Hazlitt. Read the passage carefully. Then write an essay in which you analyze the rhetorical strategies Hazlitt uses to develop his position about money.

ap lang tips for essays

Student essay example:

In his essay, Hazlitt develops his position on money through careful use of adjectives and verbs, hypothetical situations, and images. His examples serve to impress upon the reader the highly negative consequences of being in “want of money.”

Hazlitt’s word choice in his opening phrase provides an example of his technique in the rest of the essay. It is not necessary to follow “literally” with “truly” yet his repetition of the same ideas emphasizes his point. In his next sentence, one that lasts forty-six lines, Hazlitt condignly repeats similar ideas, beating into his audience the necessity of having money in this world. The parallelism throughout that one long sentence, “it is not to be sent for to court, or asked out to dinner…it is not to have your own opinion consulted or sees rejected with contempt..” ties the many different situations Haziltt gives together. What could have become a tedious spiel instead becomes a melodious recitation, each example reminding you of one before it, either because of the similarities in structure or content. Hazlitt addresses many different negative effects of not having money but manages to tie them together with his rhetorical strategies. 

The diction of the passage fully relays Hazlitt’s position about money. In every example he gives a negative situation but in most emphasizes the terrible circumstance with strong negative adjectives or verbs. “Rejected,” “contempt,” “disparaged,” “scrutinized,” “irksome,” “deprived,” “assailed” “chagrin;” the endless repetition of such discouragement shows how empathetically Hazlitt believes money is a requisite for a happy life. Even the irony of the last sentences is negative, conveying the utter hopelessness of one without money. Through one may have none in life, pitiless men will proceed to mock one’s circumstances, “at a considerable expense” after death! 

In having as the body of his essay one long sentence, Hazlitt creates a flow that speeds the passage along, hardly giving the reader time to absorb one idea before another is thrown at him. The unceasing flow is synonymous with Hazlitt’s view of the life of a person without money: he will be “jostled” through life, unable to stop and appreciate the beauty around him or to take time for his own leisure. 

The score on this essay was a 6 out of 6. This essay started out very strong as the student had a concrete thesis statement explaining the strategies that Hazlitt used to develop his position on money as well as Hazlitt’s belief on the topic. In the thesis statement, the student points out that adjectives, verbs, hypothetical situations, and images help prove Hazlitt’s point that wanting money can be problematic. 

Next, the student broke down their points into three main subsections related to their thesis. More specifically, the student first discusses word choice of repetition and parallelism. When the student discusses these strategies, they list evidence in the paragraph that can be found chronologically in Hazlitt’s essay. The next paragraph is about diction, and the student used specific adjectives and verbs that support this idea. In the last paragraph, the student emphasized how the speed and flow of the essay helped describe Hazlitt’s viewpoint on life. This last concluding sentence is particularly thoughtful, as it goes beyond the explicit points made in the essay and discusses the style and tone of the writing. 

It is important to remember that in some ways, the rhetorical essay is also an argumentative essay, as the student must prove how certain rhetorical strategies are used and their significance in the essay. The student even discussed the irony of the paragraph, which is not explicit in the passage.

Overall, this student did an excellent job organizing and structuring the essay and did a nice job using evidence to prove their points. 

Now that you’ve learned about the AP Lang rhetorical essay, you may be wondering how your AP scores impact your chances of admission. In fact, your AP scores have relatively little impact on your admissions decision , and your course rigor has much more weight in the application process.

If you’d like to know your chances of admission, be sure to check out our chancing calculator! This tool takes into account your classes, extracurriculars, demographic information, and test scores to understand your chances at admission at over 600 schools. Best of all, it is completely free!

ap lang tips for essays

Related CollegeVine Blog Posts

ap lang tips for essays

Are you seeking one-on-one college counseling and/or essay support? Limited spots are now available. Click here to learn more.

How to Write the AP Lang Synthesis Essay with Example

September 5, 2023

If you’re highly interested in learning more about writing analysis, then chances are you enrolled in AP Lang. Essentially, AP Lang is an advanced course for high schoolers that combines interest and knowledge in English with critical thinking. In the class, students learn how to analyze and synthesize a variety of texts to construct well-reasoned arguments. If you take AP Lang, then you can opt to take the AP test at the conclusion of the school year. On the exam, students write the AP Lang synthesis essay to demonstrate their learned abilities. In this article, we’ll look at what the AP Lang synthesis essay requires and show an example to provide better understanding of what to expect on the exam.

AP Lang Exam Basics

The AP Lang exam is separated into two sections. In the first section, students have one hour to answer a series of 45 multiple-choice questions. Here, about half of the questions are based on passages students read. The other half are focused on the best revision techniques. Essentially, the answers for the latter 20-22 questions are geared toward revising mock essays.

In this article, however, we’ll focus mainly on the second part of the exam: the AP Lang synthesis essay.

In this second section, students have two hours and 15 minutes to write three essays of their own design. The three open-ended questions in this section are intended to be free-response and allow for a variety of approaches. Each question is intended to allow up to 40 minutes to complete.

For the AP Lang synthesis essay, students are presented with a scenario of the College Board’s design. The scenario will provide its own thesis statement. Usually, scenarios relate to real-world problems like environmental concerns, media, or government policies.

For each scenario, students are provided with 6-7 outside sources. These sources could be in the form of an image, visual graph, or written paragraph. For written paragraphs, the sources are usually no more than 500 words.

Students are then expected to incorporate at least 3-4 of these outside sources into their essay response. The outside sources are intended to be used as supporting evidence for the student’s chosen stance or argument. Students are able to either agree with or disagree with the thesis presented in the original scenario.

AP Lang Exam – Scoring

In the second part of the AP Lang exam, students can earn a possible 6 points on each essay. 1 point is earned for the development of a thesis. Up to 4 points can be earned for evidence and commentary. The final 1 point is earned for sophistication of thought.

AP Lang Exam – Takeaways

Ultimately, the goal of the AP Lang synthesis essay is not whether the student is “right” or “wrong” in their argument. The key is that students are able to reasonably and clearly support their argument using the provided sources as evidence .

The College Board looks for your ability to identify relationships between texts , form a coherent argument , and interpret external sources .

Synthesis Essay AP Lang Examples

If you’re not sure how the questions will look on the AP Lang synthesis essay section, we’ll provide an example. After the example, we’ll break down the strengths and weaknesses of the response. That way, you’ll have a better idea of what the College Board is looking for.

Additionally, the College Board has released previous AP Lang synthesis essay examples you can review. They even have essay questions as recent as 2022 . For further support, a scoring commentary and comments from the Chief Reader are also available to view. Additionally, there are other examples you can view from earlier years .

Note: A good strategy to study for the synthesis essay AP Lang exam is to review your rhetorical devices and literary devices . Understanding how these devices function can be essential in constructing a cohesive essay.

Synthesis Essay AP Lang Examples – Sample Question

Below is a sample question from the AP Lang synthesis essay and a response to the prompt. This question was taken directly from a 2022 exam . However, the response to the question will be originally crafted for the purpose of this newsletter. As well, all supporting evidence will be originally created and does not correspond to any previous test.

The Question

Since the early 2000s, the United States government and a number of corporations have sponsored initiatives to improve education in the STEM disciplines: science, technology, engineering, and mathematics. The emphasis on STEM subjects in elementary, secondary, and higher education reflects concerns that United States students are less proficient in these areas than are students in other countries. Additionally, there is a belief that mastery in STEM fields is now essential in order to join a highly technical and specialized workforce. However, not everyone is convinced that a STEM-focused curriculum is necessary and/or effective.

In your response you should do the following:

  • Respond to the prompt with a thesis that presents a defensible
  • Select and use evidence from at least three of the provided sources to support your line of Indicate clearly the sources used through direct quotation, paraphrase, or summary. Sources may be cited as Source A, Source B, etc., or by using the description in parentheses.
  • Explain how the evidence supports your line of
  • Use appropriate grammar and punctuation in communicating your

How to Approach the Question

Maybe your first thought upon seeing this block of text is to feel overwhelmed. But don’t panic. There are effective ways to approach the question so you will be more prepared in your response.

It’s a good strategy to first isolate the thesis . What is the main idea of the text, and what is its argument?

Try it out. Reread the prompt and see if you can identify what the statement is asking you to develop an opinion on.

Think you’ve got it? In this example, we will be focusing on whether or not a STEM-focused curriculum in K-12 education is necessary and/or effective. In short, we will be arguing either for (highlighting the benefits) or against (highlighting the pitfalls) a STEM-focused curriculum.

How do we know what this statement is asking us?

Well, the statement provides a lot of background information. For example, we receive a definition of what STEM stands for. As well, we know that since 2000, there has been a greater initiative for STEM-focused classes.

When you read the prompt for the first time, it’s a great strategy to learn how to differentiate between background and contextual information from the heart of the argument .

A good way to learn how to isolate the argument is to look for transition words. Usually, these appear near the end of the question. Words like “however” and “yet” are signals that the statement is offering a differing opinion. Typically, the statement will tell you which two positions it’s offering for argument. These opinions are usually signaled by contrasting transition words.

So, now that we know what the question is asking us, what is the best way to respond?

Synthesis Essay AP Lang Examples – Sample Answer

The following is an essay response I crafted to the above question. After reading the sample, I will break down what it does well and what areas can be improved.

A STEM-focused curriculum is not as essential to providing a meaningful K-12 education. Because the majority of high school students are not proficient in STEM-focused classes, prioritizing these classes causes harm to student’s mental health and academic performance.

As seen in Source A, 60% of high school seniors in the Midwest only scored a C average in math and science-based classes (Langston). This statistic suggests that the majority of students do not resonate with STEM classes and therefore perform poorly. Earning a low score in any class does not bode well for students’ mental health.

When looking at the primary argument in Source C, it’s clear that most high schoolers prefer creative outlets to fact-based research (Kohler). Allowing students the opportunity to be more creative and initiate conversations about coursework lets students be more active in their learning. When students can discuss the nuance in their opinions, more personal growth happens. These conversations are not always easy to have in STEM-focused classes.

As well, when looking back to Source A, it’s clear that high school students in the Midwest earned higher grades, on average, in their English and art classes (Langston). This figure suggests that students perform better in these classes because they relate more to the source material. When relating to what they learn, they perform better in class.

In conclusion, STEM-focused curriculum is not as essential in K-12 education because most high school students do not relate to their STEM classes. When students do not earn satisfactory grades in these classes, it negatively affects their future college applications and job prospects.

Synthesis Essay AP Lang Examples – Answer Breakdown

So, what does this essay response get right, and where can it be improved? Let’s start with what the response does well.

First, the response establishes its thesis right away. Usually, it’s a good idea to clearly state your argument within the first paragraph. Not only is this a good practice because a reader can easily identify your stance, but also you can refer to your thesis as you write to make you stay on track.

With your thesis, it’s also a good idea to include one to two supporting sentences with the reasons why the thesis is concluded . Like in this example, I wrote that STEM-focused classes should not be prioritized because they can negatively affect both mental health and academic performance.

Another positive aspect of this response is that it is sure to not only reference but also cites its sources . It’s important that the reader understand where your information is coming from. That way, the readers can ensure you are interpreting the sources correctly.

AP Lang Synthesis Essay (Continued)

However, when rereading the instructions, it’s clear that this response fails the basic requirement of referring to at least three sources. Always make sure to reread the instructions to ensure you meet the standard requirements for incorporating source material.

Further, this AP Lang synthesis essay does not fully support its arguments . Ideas are simply stated and are not expanded upon.

For example, I mentioned a few times that earning low grades in STEM classes leads to negative mental health for high school students. However, there is no source referenced that either confirms or denies this claim. Therefore, there is no sufficient evidence to support my argument. It relies purely on inference.

Additionally, this AP Lang synthesis essay does not arrive at a sufficient level of sophistication of thought . Basically, sophistication of thought means avoiding broad generalizations and vague claims. The more specific you can be, the better your argument will sound.

Synthesis Essay AP Lang – In Conclusion

In the end, it’s always helpful to read the prompt thoroughly before writing. As well, making notes while you read could be a good strategy to pinpoint main ideas both in the prompt and the sources. That way, you can reread the material quickly. Similarly, sketching an outline may also be helpful. In addition, you should always carefully read the instructions to ensure all guidelines are followed.

As long as you avoid broad generalizations and use enough supporting evidence for your claim, you will be on the right path!

  • High School Success

Meghan Dairaghi

With a BA in English and an MFA in Creative Writing, Meghan has served as a writing tutor at the University of Missouri St. Louis and Maryville University. Additionally, Meghan has held editorial roles at River Styx and Boulevard, and was a prose reader at Farside Review . Most recently, her work has been featured in Belle Ombre , Flypaper Lit , and Mag 20/20 , among others, and she was nominated for the Mary Troy Prize in Fiction. 

  • 2-Year Colleges
  • ADHD/LD/Autism/Executive Functioning
  • Application Strategies
  • Best Colleges by Major
  • Best Colleges by State
  • Big Picture
  • Career & Personality Assessment
  • College Essay
  • College Search/Knowledge
  • College Success
  • Costs & Financial Aid
  • Data Visualizations
  • Dental School Admissions
  • Extracurricular Activities
  • Graduate School Admissions
  • High Schools
  • Homeschool Resources
  • Law School Admissions
  • Medical School Admissions
  • Navigating the Admissions Process
  • Online Learning
  • Outdoor Adventure
  • Private High School Spotlight
  • Research Programs
  • Summer Program Spotlight
  • Summer Programs
  • Teacher Tools
  • Test Prep Provider Spotlight

“Innovative and invaluable…use this book as your college lifeline.”

— Lynn O'Shaughnessy

Nationally Recognized College Expert

College Planning in Your Inbox

Join our information-packed monthly newsletter.

Get the Reddit app

No matter what course you are taking, we are a community that helps students earn college credit!

AP Lang exam tips you may not have heard

Hello everybody! The UWorld English team is a group of former AP teachers and AP readers. We talked together and came up with this short list of tips that might help you for your exam.

Reading MCQs

People think the answers are subjective; they're not. Right answers are completely supported by evidence, while wrong answers are sort of supported or not at all.

Don't waste time studying a bunch of terms. Instead, make sure you know these words: exigence, qualify/qualification, ambiguous, underscore, and undermine (not the same thing).

Answer questions about specific lines/paragraphs first. Answer questions about the overall passage last.

Pay attention to how the questions/ answers actually help you understand the passage.

Writing MCQs

You got these. Fuhgeddaboudit. But if you do need help, think about these ideas.

Biggest tip: Writing MCQs are really reading questions in disguise. Read the whole draft.

You can get away with not reading the writing passages on the SAT, but not here. They won't ask about punctuation or grammar; they are primarily asking about coherency and support. That means you have to know what direction the draft is going so you can choose the best organization and support for its claims.

Transition questions: Identify what the information after the transition does (3 basic categories). Does it...

contrast the info that came before it?

give more of the same as what came before it?

clarify the information that came before?

Choose the transition word/phrase that accurately communicates what the info does.

Style questions: To choose the best word or to decide whether to keep/delete a word, consider these two things:

How formal or casual is the language in the draft?

Is the tone more positive or negative?

The right answer will be as formal/casual as the rest of the language and will likely match the positivity/negativity of the rest of the draft.

Bias questions: Pick the answer that has no bias (prejudice).

Look at connotations and pick the answer that's most neutral. (eg. Calling someone a "health nut" makes them sound a little crazy = biased. Saying someone is a "health-conscious consumer" is neither good nor bad = unbiased.)

Crediting a source questions : Pick the most specific answer.

The right answer gives specific info about who/what the source is and why it can be trusted. (eg. My dad says he makes the best cookies. = So? Prove it. My dad, Tim Casey, the CEO of Mrs. Fields' Cookies, says he makes the best cookies. = Ok, he's got cred.)

Add/Keep/Delete questions : Whether a sentence stays or goes, it's all about picking the right reason for it.

Decide if the info in question actually does what the answer choice claims. (eg. Keep it, because it provides evidence from a scientific study ( does it? ) that supports the claim that ants are more intelligent than humans ( does the evidence actually support that claim? )

Synthesis essay

Easiest essay IMO. Don't read all 6-7 sources. No need.

This essay, like the Arg essay, will use a topic that there's not a clear yes/no, good/bad opinion on. It will have complexity; everybody's perspective will have something positive and negative about it. Acknowledge that, but write in a more opinionated way about one perspective than the other = 70% your opinion, 30% opposition's point of view. Do NOT write a 50/50 essay.

Pick 2 pieces of evidence to support your argument and 1 from the opposition to use as a concession or to refute. That gets you points for using 3 pieces of evidence and lets you write a couple of good body paragraphs.

Rhetorical analysis essay

If the passage is more modern, it's the second easiest essay.

Consider these questions: How does the speaker make the topic sound? How did you know? That's really the crux of the entire essay.

Remember, nearly anything a writer does in the text can be a strategy, and it's ok if there's not an official "literary" name for it. For example, using we/us = unifying language. Talking about his background = building credibility. Shaming the audience = guilt trip. The first paragraph is a good place to look for strategies because writers have to work hard to engage their audience.

Top essay score is a 6. You can make an easy 4. Here's how:

ALWAYS write a thesis = 1 point.

The most common point split for Evidence and Commentary is between a 2 and 3. Don't summarize the passage; that only earns you a point. Pick something specific the speaker does to get his purpose across and explain how the speaker makes it work = 2 points. Do it again = 3 points. Anything beyond that is gravy.

Don't worry about the Sophistication point. It will either come out of your pen or it won't.

Argument essay

Hardest one. If you're stuck for ideas, try using the "ripple effect." Start close and venture out. Think of a personal example, an example from someone you know/ something you've seen, then something/someone famous, influential, or historical. Valid examples can come from art, science, sports, music, gaming, and entertainment. They don't always have to come from a textbook.

Same general tips apply as in the RA essay. Must write thesis = 1 point. Explain how one example proves your point = 2 points. Explain how two examples prove your point = 3 points. More than two examples = o7

Caution: Don't get lost in telling all the details of your example. Just focus on the parts that tie into the point you're making and spend your energy explaining how your example proves it.

Remember that the best thing you can do for your MCQ score is to practice questions with comprehensive explanations. We have hundreds of questions like this here .

Good luck on your studies!

By continuing, you agree to our User Agreement and acknowledge that you understand the Privacy Policy .

Enter the 6-digit code from your authenticator app

You’ve set up two-factor authentication for this account.

Enter a 6-digit backup code

Create your username and password.

Reddit is anonymous, so your username is what you’ll go by here. Choose wisely—because once you get a name, you can’t change it.

Reset your password

Enter your email address or username and we’ll send you a link to reset your password

Check your inbox

An email with a link to reset your password was sent to the email address associated with your account

Choose a Reddit account to continue

logo-type-white

AP® English Language

3 ap® english language rhetorical essay strategies.

  • The Albert Team
  • Last Updated On: March 1, 2022

3_ap_english_language rhetorical essay strategies

The AP® English Language rhetorical essay can be nightmare inducing for some AP® students, but there is no need for fear. In this exam review we will lay out helpful strategies to get you through the rhetorical essays in no time.

Rhetorical Strategy #1: Dissecting the Prompt

The first rhetorical essay strategy is to dissect the prompt. Understanding what the rhetorical essay wants from you is essential. It is important for you to read the prompt carefully for every essay, but critical reading is even more essential to the rhetorical essay. Your rhetorical prompt that you will be given for the AP® English Language exam will contain two elements. The first element is the concrete task that the prompt is asking of you, which is always to analyze the passage that follows. The second part of the prompt is a more abstract task that is not directly asked for in the prompt, but it is implied. By completely understanding both parts of the prompt, you will be able to give a complete essay that will get you to a higher score.

One example of a prompt from an AP® English Language rhetorical essay is this one from the 2008 exam . The prompt reads:

“In the following passage from The Great Influenza, an account of the 1918 flu epidemic, author John M. Barry writes about scientists and their research. Read the passage carefully. Then, in a well-written essay, analyze how Barry uses rhetorical strategies.”

Here you can see the concrete task that the examiners are asking. They want you to analyze the passage for rhetorical strategies; however, you must figure out what you are analyzing the passage for. That is the more abstract concept that you need to dissect the prompt to find. In the case of Barry’s passage you will need to analyze how he uses rhetorical strategies in order to portray scientific research. We know this, because if you look at the prompt, it specifically states what Barry did in his work, which was to write about science and research. That is your abstract task.

Once you have found your concrete task and your abstract task, a great strategy is to write it down to keep you focused throughout your essay . Using the example above this would look like the following:

Analyze how Barry uses rhetorical strategies in order to portray scientific research.

That sentence is what you must follow when writing your essay, and if you successfully keep to this task, then you will move closer to that high score.

Rhetorical Essay Strategy #2: Stick to the Format

This next rhetorical essay strategy is the key to great organization and structure that will put your test anxiety to bed. There is a simple paragraph structure for the body paragraphs of the AP® English Language rhetorical essay that will allow you to think, write, and score higher, faster. You need to begin each body paragraph with an assertion or claim. That is the point that you are trying to make clear to your audience what you will be proving. A great example of this is from the 2006 AP® English Language rhetorical essay. Below is student 2B’s opening sentence for her first body paragraph.

“The diction of the passage fully relays Hazlitt’s position about money ( student 2B ).”

You can see how the student directly asserts what he or she will be proving in this statement. The next step in constructing your body paragraph is to give one to two pieces of textual evidence. Be sure to state why these quotations relate back to your claim, otherwise they will be deemed irrelevant by the examiners. An example of this is the next sentence in student 2B’s body paragraph about diction. Here, the student brings in elements from the text to support his or her claim about Hazlitt using diction.

“’Rejected’, ‘contempt’, ‘disparaged’, ‘scrutinized’, ‘irksome’, ‘deprived’, ‘assailed’, ‘chagrin’; the endless repetition of such discouragement shows just how emphatically Hazlitt money is requisite for happy life (student 2B).”

The final part of this strategy for conquering the body paragraphs of your rhetorical essays is to end those body paragraphs with a thorough analysis. This is the aspect of the exam where you can put your way of looking at the text into your essay.

An example of this is at the end of student 2B’s body paragraph where he or she states, “The irony of the last sentences is negative, conveying the utter hopelessness of one without money. Though one may have none in life, pitiless men will continue to mock one’s circumstances even after death! (student 2B)”

This analysis of the text adds to the textual examples above and continues to bring in new logic from the student.

When this format of a body paragraph is followed, then it is extremely effective. The essay becomes clear, assertive, and easy to follow for the examiners. Follow this rhetorical essay strategy and you are even closer to getting that 5 on the exam.

Rhetorical Essay Strategy #3: LORA

As you are looking at your AP® English Language rhetorical essay prompt and passage it is important to remember the mnemonic device, LORA. LORA stands for Language, Organization, and Rhetorical Appeals. These elements will help you form your argument.

When you read through your passage you want to think about how the author is utilizing language. Is he or she using figurative language effectively? Is there imagery within the passage? Does the diction of the passage make it more rhetorically persuasive? You should not use all of these, but picking one and analyzing it clearly in one paragraph will keep you focused on how the author uses rhetoric, which is the main task of this essay.

An example of this was in the 2006 AP® English Language rhetorical essay. Student 2A begins his or her first body paragraph with, “One of Hazlitt’s most effective methods of promoting the importance of money is his strong diction (student 2A).” This student begins his or her essay with focusing on diction as how the language is used. He or she then goes on to explain why diction betters Hazlitt’s argument, which is exactly what you must do for your own rhetorical essay.

The organization of the author is the next part of your answer to the prompt. You want to look at how the author organized his or her ideas within the passage to support his or her own argument. By pointing out the organization, or structure, of the work and how it adds to the overall persuasiveness, you will bring two of the three most important elements of rhetoric together in your essay.

After organization you need to look at the rhetoric appeals. You may know them by the names logos, pathos, and ethos. It is suggested that you cover as many of these as possible; however, if time does not permit or if the passage uses one more than the other, then you should focus on one appeal.

One example of using pathos in an essay is from student 2A from the 2006 prompt. “Hazlitt plays on the audience’s heartstrings for more than enough time to convince them of the importance of having money (student 2A).” While it would have been better for the student to directly say that this is pathos, he or she does thoroughly explain the appeal to the passions, or pathos.

Key Takeaways

When taking the AP® English Language rhetoric essay you just need to remember these three rhetorical essay strategies: dissect the prompt, follow the format, and always include LORA. If you can follow them, then you are already on your way to a 5 on the AP® English Language exam .

Let’s put everything into practice. Try this AP® English Language practice question:

Rhetorical Considerations AP® English Language Practice Question

Looking for more AP® English Language practice?

Check out our other articles on AP® English Language .

You can also find thousands of practice questions on Albert.io. Albert.io lets you customize your learning experience to target practice where you need the most help. We’ll give you challenging practice questions to help you achieve mastery of AP® English Language.

Start practicing here .

Are you a teacher or administrator interested in boosting AP® English Language student outcomes?

Learn more about our school licenses here .

Interested in a school license?​

Popular posts.

AP® Physics I score calculator

AP® Score Calculators

Simulate how different MCQ and FRQ scores translate into AP® scores

ap lang tips for essays

AP® Review Guides

The ultimate review guides for AP® subjects to help you plan and structure your prep.

ap lang tips for essays

Core Subject Review Guides

Review the most important topics in Physics and Algebra 1 .

ap lang tips for essays

SAT® Score Calculator

See how scores on each section impacts your overall SAT® score

ap lang tips for essays

ACT® Score Calculator

See how scores on each section impacts your overall ACT® score

ap lang tips for essays

Grammar Review Hub

Comprehensive review of grammar skills

ap lang tips for essays

AP® Posters

Download updated posters summarizing the main topics and structure for each AP® exam.

COMMENTS

  1. How to Write the AP Lang Argument Essay (With Example)

    Typically, the AP Lang Argument Essay prompt asks you to reflect on a broad cultural, moral, or social issue that is open to debate. For evidence, you won't be asked to memorize and cite statistics or facts. Rather, you'll want to bring in real-world examples of: Historical events. Current-day events from the news.

  2. How to Write the AP Lang Argument Essay + Examples

    2. Pick one side of the argument, but acknowledge the other side. When you write the essay, it's best if you pick one side of the debate and stick with it for the entire essay. All your evidence should be in support of that one side. However, in your introductory paragraph, as you introduce the debate, be sure to mention any merit the ...

  3. The Ultimate List of AP® English Language Tips

    The AP® Language and Composition exam tests your ability to not only read content, but also to analyze what you have read and draw conclusions to present in an argument. The test consists of two parts: multiple choice and free response, and it lasts 3 hours and 15 minutes. It is a long and difficult test, but with the right amount of practice ...

  4. How to Write the AP Lang Rhetorical Analysis Essay (With Example)

    The AP Lang Rhetorical Analysis Essay is one of three essays included in the written portion of the AP English Exam. The full AP English Exam is 3 hours and 15 minutes long, with the first 60 minutes dedicated to multiple-choice questions. Once you complete the multiple-choice section, you move on to three equally weighted essays that ask you ...

  5. How to Write the AP Lang Synthesis Essay + Example

    The argumentative essay requires you to pick a perspective of a debate and create an argument based on the evidence provided. In this post, we will take a look at the AP Lang synthesis essay and discuss tips and tricks to master this part of the exam. We will also provide an example of a well-written essay for review.

  6. Ultimate Guide to the AP English Language and Composition Exam

    AP Language and Composition Score Distribution, Average Score, and Passing Rate. In 2019, 54.3% of the students who took the AP English Language and Composition exam received a score of 3 or higher. Only 9.9% of students who took the exam achieved the top score of 5, and 14.5% of students who took the exam scored a 1.

  7. How to Ace the AP Language Argument Essay

    Learn how to write the AP Language & Composition argument essay step by step in this tutorial! I'll show you the prompt from the 2018 exam and guide you thro...

  8. The Best AP® English Language Review Guide for 2024

    5 AP® English Language and Composition FRQ Study Tips. 1. Practice answering questions from The College Board's archive of past exam questions. Typically, the same skills are assessed from year to year, so practicing with released exams is a great way to brush up on your analysis skills. 2.

  9. 5 Tips to Improve Your Argument Essay

    This video offers tips about how to write an effective argument essay for the AP Lang exam.Ebook Info:Want all of Coach Hall's best tips for rhetorical analy...

  10. Crafting an Impressive Argumentative Essay for AP Lang

    Writing an argumentative essay for AP Language and Composition requires a strategic approach to effectively convey your perspective. Here's a guide to crafting an impressive argumentative essay: 1. Understand the Prompt: - Tip: Carefully read and analyze the prompt. Identify the key elements, including the task you are asked to perform and any ...

  11. AP Lang Exam Guide

    Format of the 2024 AP English Language and Composition exam. This year, all AP exams will cover all units and essay types. The 2024 AP English Language and Composition exam format will be: Section I: Multiple Choice - 45% of your score- - 45 questions in 1 hour. Section II: Free Response Section - 55% of your score- - 2 hours and 15 minutes for ...

  12. How to Write a Perfect Synthesis Essay for the AP Language Test

    Paragraph 1: The prompt presents and briefly explains the topic that you'll be writing your synthesis essay about. That topic is the concept of eminent domain. Paragraph 2: The prompt presents a specific claim about the concept of eminent domain in this paragraph: Eminent domain is productive and beneficial.This paragraph instructs you to decide whether you want to defend, challenge, or ...

  13. Expert Guide to the AP Language and Composition Exam

    The AP English Language and Composition Multiple-Choice. The multiple-choice section tests you on two main areas. The first is how well you can read and understand nonfiction passages for their use of rhetorical devices and tools. The second is how well you can "think like a writer" and make revisions to texts in composition questions.

  14. Guide to the AP English Language and Composition Exam

    Argument essay: Presents a claim or assertion in the prompt and then asks you to argue a position based on your own knowledge, experience, or reading. How to Interpret AP English Language Scores . AP scores are reported from 1 to 5. Colleges are generally looking for a 4 or 5 on the AP English Language exam, but some may grant AP credit for a 3 ...

  15. How to Ace the AP Language Rhetorical Analysis Essay

    In this video, I'll show you how to write the AP English Language rhetorical analysis essay (Q2) step by step using the actual 2017 prompt. Watch me annotate...

  16. How to Get a 6 on Argument FRQ in AP® English Language

    Pick an opinion and stick to it. Choose one side of the argument and one clear claim to support all the way through. Craft a thesis statement. Your thesis should be clear, concise, and introduce the content of your essay. Craft a chronological argument. Make an argument that builds on its prior points.

  17. AP English Language and Composition

    The AP English Language and Composition Exam has question types and point values that stay consistent from year to year, so you and your students know what to expect on exam day. ... 20-22 Writing questions that ask students to "read like a writer" and consider revisions to stimulus texts. Section II: Free Response. 3 Questions | 2 hours ...

  18. Expert Guide to the AP Language and Composition Exam

    1. Exam Format: The AP Language and Composition Exam consists of multiple-choice questions and free-response tasks. The multiple-choice section tests your reading comprehension and analysis skills, while the free-response section assesses your ability to write coherent and persuasive essays. 2.

  19. How to Write the AP Lang Rhetorical Essay

    Tips for Writing the AP Lang Rhetorical Essay. 1. Outline Your Essay Before Writing. One of the most important parts of the AP Lang essays is structuring your essay so that it makes sense to the reader. This is just as important as having good content. For this essay in particular, you'll want to read the passage first and write a brief ...

  20. How to Write the AP Lang Synthesis Essay with Example

    AP Lang Exam Basics. The AP Lang exam is separated into two sections. In the first section, students have one hour to answer a series of 45 multiple-choice questions. Here, about half of the questions are based on passages students read. The other half are focused on the best revision techniques.

  21. AP Lang exam tips you may not have heard : r/APStudents

    ADMIN MOD. AP Lang exam tips you may not have heard. Hello everybody! The UWorld English team is a group of former AP teachers and AP readers. We talked together and came up with this short list of tips that might help you for your exam. Reading MCQs. People think the answers are subjective; they're not. Right answers are completely supported ...

  22. 11 AP® English Language Test Taking Strategies

    There are strategies that will greatly increase your ability to excel on multiple choice exams; however, the best strategy is to practice frequently. Taking practice tests months before the AP® English Language exam will get your brain ready and help you to train yourself how to think. 9. The Art of Guessing.

  23. 3 AP® English Language Rhetorical Essay Strategies

    The AP® English Language rhetorical essay can be nightmare inducing for some AP® students, but there is no need for fear. In this exam review we will lay out helpful strategies to get you through the rhetorical essays in no time. Rhetorical Strategy #1: Dissecting the Prompt. The first rhetorical essay strategy is to dissect the prompt.